Difference between revisions of "2021 AMC 12A Problems/Problem 11"

m (Problem)
(Solution 4 (System of linear equations))
 
(68 intermediate revisions by 3 users not shown)
Line 5: Line 5:
  
 
==Diagram==
 
==Diagram==
[[File:2021 AMC 12A Problem 11 (1) LaTeX Revised.png|center]]
+
<asy>
Graph in Desmos: https://www.desmos.com/calculator/bsiulzrjrn
+
/* Made by MRENTHUSIASM */
 +
size(200);
  
 +
int xMin = -3;
 +
int xMax = 9;
 +
int yMin = -3;
 +
int yMax = 7;
 +
 +
draw((xMin,0)--(xMax,0),black+linewidth(1.5),EndArrow(5));
 +
draw((0,yMin)--(0,yMax),black+linewidth(1.5),EndArrow(5));
 +
label("$x$",(xMax,0),(2,0));
 +
label("$y$",(0,yMax),(0,2));
 +
 +
pair A = (3,5);
 +
pair B = (0,2);
 +
pair C = (2,0);
 +
pair D = (7,5);
 +
 +
draw(A--B--C--D,red);
 +
dot(A,linewidth(3.5));
 +
dot(B,linewidth(3.5));
 +
dot(C,linewidth(3.5));
 +
dot(D,linewidth(3.5));
 +
label("$(3,5)$",A,(0,2));
 +
label("$(7,5)$",D,(0,2));
 +
</asy>
 
~MRENTHUSIASM
 
~MRENTHUSIASM
  
==Solution 1==
+
==Solution 1 (Reflections)==
Every time the laser bounces off a wall, instead we can imagine it going straight by reflecting it about the wall. Thus, the laser starts at <math>(3, 5)</math> and ends at <math>(-7, -5)</math>, so the path's length is <math>\sqrt{10^2+10^2}=\boxed{\textbf{(C)} 10\sqrt{2}}</math>
+
Let <math>A=(3,5)</math> and <math>D=(7,5).</math> Suppose that the beam hits and bounces off the <math>y</math>-axis at <math>B,</math> then hits and bounces off the <math>x</math>-axis at <math>C.</math>
~JHawk0224
 
  
==Solution 2 (Detailed Explanation of Solution 1)==
+
When the beam hits and bounces off a coordinate axis, the angle of incidence and the angle of reflection are congruent. Therefore, we straighten up the path of the beam by reflections:
Let <math>A=(3,5), D=(7,5), B</math> be the point where the beam hits the <math>y</math>-axis, and <math>C</math> be the point where the beam hits the <math>x</math>-axis.
+
<ol style="margin-left: 1.5em;">
 +
  <li>We reflect <math>\overline{BC}</math> about the <math>y</math>-axis to get <math>\overline{BC'}.</math></li><p>
 +
  <li>We reflect <math>\overline{CD}</math> about the <math>x</math>-axis to get <math>\overline{C'D'}</math> with <math>D'=(7,-5),</math> then reflect <math>\overline{C'D'}</math> about the <math>y</math>-axis to get <math>\overline{C'D''}</math> with <math>D''=(-7,-5).</math></li><p>
 +
</ol>
 +
We obtain the following diagram:
 +
<asy>
 +
/* Made by MRENTHUSIASM */
 +
size(225);
  
Reflecting <math>\overline{BC}</math> about the <math>y</math>-axis gives <math>\overline{BC'}.</math> Then, reflecting <math>\overline{CD}</math> about the <math>y</math>-axis gives <math>\overline{C'D'}.</math> Finally, reflecting <math>\overline{C'D'}</math> about the <math>x</math>-axis gives <math>\overline{C'D''},</math> as shown below.
+
int xMin = -9;
 +
int xMax = 9;
 +
int yMin = -7;
 +
int yMax = 7;
  
[[File:2021 AMC 12A Problem 11 (2) LaTeX.png|center]]
+
draw((xMin,0)--(xMax,0),black+linewidth(1.5),EndArrow(5));
 +
draw((0,yMin)--(0,yMax),black+linewidth(1.5),EndArrow(5));
 +
label("$x$",(xMax,0),(2,0));
 +
label("$y$",(0,yMax),(0,2));
  
Graph in Desmos: https://www.desmos.com/calculator/lxjt0ewbou
+
pair A = (3,5);
 +
pair B = (0,2);
 +
pair C = (2,0);
 +
pair D = (7,5);
 +
pair E = (-2,0);
 +
pair F = (7,-5);
 +
pair G = (-7,-5);
  
It follows that <math>D''=(-7,-5).</math> The total distance that the beam will travel is  
+
draw(A--B--C--D,red);
 +
draw(B--E,heavygreen+dashed);
 +
draw(C--F,heavygreen+dashed);
 +
draw(E--G,heavygreen+dashed);
 +
dot("$A(3,5)$",A,(0,2),linewidth(3.5));
 +
dot("$B$",B,(-2,0),linewidth(3.5));
 +
dot("$C$",C,(0,-2),linewidth(3.5));
 +
dot("$D(7,5)$",D,(0,2),linewidth(3.5));
 +
dot("$C'$",E,(0,-2),linewidth(3.5));
 +
dot("$D'(7,-5)$",F,(0,-2),linewidth(3.5));
 +
dot("$D''(-7,-5)$",G,(0,-2),linewidth(3.5));
 +
</asy>
 +
The total distance that the beam will travel is  
 
<cmath>\begin{align*}
 
<cmath>\begin{align*}
AB+BC+CD&=AB+BC'+C'D' \\
+
AB+BC+CD&=AB+BC+CD' \\
 
&=AB+BC'+C'D'' \\
 
&=AB+BC'+C'D'' \\
 
&=AD'' \\
 
&=AD'' \\
 
&=\sqrt{((3-(-7))^2+(5-(-5))^2} \\
 
&=\sqrt{((3-(-7))^2+(5-(-5))^2} \\
&=\sqrt{200} \\
 
 
&=\boxed{\textbf{(C) }10\sqrt2}.
 
&=\boxed{\textbf{(C) }10\sqrt2}.
 
\end{align*}</cmath>
 
\end{align*}</cmath>
 +
~MRENTHUSIASM (Solution)
 +
 +
~JHawk0224 (Proposal)
 +
 +
==Solution 2 (Parallelogram)==
 +
Define points <math>A,B,C,</math> and <math>D</math> as Solution 1 does. Moreover, let <math>E</math> be a point on <math>\overline{CD}</math> such that <math>\overline{BE}</math> is perpendicular to the <math>y</math>-axis, and <math>F</math> be a point on <math>\overline{BE}</math> such that <math>\overline{CF}</math> is perpendicular to the <math>x</math>-axis, as shown below.
 +
<asy>
 +
/* Made by MRENTHUSIASM */
 +
size(200);
 +
 +
int xMin = -3;
 +
int xMax = 9;
 +
int yMin = -3;
 +
int yMax = 7;
 +
 +
draw((xMin,0)--(xMax,0),black+linewidth(1.5),EndArrow(5));
 +
draw((0,yMin)--(0,yMax),black+linewidth(1.5),EndArrow(5));
 +
label("$x$",(xMax,0),(2,0));
 +
label("$y$",(0,yMax),(0,2));
 +
 +
pair A = (3,5);
 +
pair B = (0,2);
 +
pair C = (2,0);
 +
pair D = (7,5);
 +
pair E = (4,2);
 +
pair F = (2,2);
 +
 +
draw(A--B--C--D,red);
 +
draw(A--D^^B--E^^C--F,heavygreen+dashed);
 +
dot("$A(3,5)$",A,(0,2),linewidth(3.5));
 +
dot("$B$",B,(-2,0),linewidth(3.5));
 +
dot("$C$",C,(0,-2),linewidth(3.5));
 +
dot("$D(7,5)$",D,(0,2),linewidth(3.5));
 +
dot("$E$",E,(2,0),linewidth(3.5));
 +
dot("$F$",F,(0,1.5),linewidth(3.5));
 +
</asy>
 +
When the beam hits and bounces off a coordinate axis, the angle of incidence and the angle of reflection are congruent, from which <math>\angle ABF=\angle CBF</math> and <math>\angle BCF=\angle ECF.</math> We conclude that <math>\triangle BCF\cong\triangle ECF</math> by ASA, so <math>\angle CBF=\angle CEF.</math> It follows that <math>\angle ABF=\angle CEF</math> by transitive, so <math>\overline{AB}\parallel\overline{CD}</math> by the Converse of the Alternate Interior Angles Theorem.
 +
 +
Note that <math>\overline{AD}\parallel\overline{BE}.</math> Since the opposite sides are parallel, quadrilateral <math>ABED</math> is a parallelogram. From <math>\triangle BCF\cong\triangle ECF,</math> we get <math>BF=EF=2,</math> so <math>C=(2,0).</math>
 +
 +
Let <math>B=(0,b).</math> We equate the slopes of <math>\overline{AB}</math> and <math>\overline{DC}:</math> <cmath>\frac{5-b}{3-0}=\frac{5-0}{7-2},</cmath> from which <math>b=2,</math> or <math>B=(0,2).</math>
 +
 +
By the Distance Formula, we have <math>AB=3\sqrt2,BC=2\sqrt2,</math> and <math>CD=5\sqrt2.</math> The total distance that the beam will travel is <cmath>AB+BC+CD=\boxed{\textbf{(C) }10\sqrt2}.</cmath>
 +
 +
<u><b>Remark</b></u>
 +
 +
When a straight line hits and bounces off a coordinate axis at point <math>P,</math> the ray entering <math>P</math> and the ray leaving <math>P</math> always have negative slopes. In this problem, <math>\overline{AB}</math> and <math>\overline{BC}</math> have negative slopes; <math>\overline{BC}</math> and <math>\overline{CD}</math> have negative slopes. So, <math>\overline{AB}</math> and <math>\overline{CD}</math> have the same slope, or <math>\overline{AB}\parallel\overline{CD}.</math>
  
 
~MRENTHUSIASM
 
~MRENTHUSIASM
  
==Solution 3 (Slopes and Parallelogram)==
+
==Solution 3 (Educated Guess)==
Define points <math>A,B,C,</math> and <math>D</math> as Solution 2 does.
+
Define points <math>A,B,C,</math> and <math>D</math> as Solution 1 does.
 +
 
 +
Since choices <math>\textbf{(B)}, \textbf{(C)},</math> and <math>\textbf{(D)}</math> all involve <math>\sqrt2,</math> we suspect that one of them is the correct answer. We take a guess in faith that <math>\overline{AB},\overline{BC},</math> and <math>\overline{CD}</math> all form <math>45^\circ</math> angles with the coordinate axes, from which <math>B=(0,2)</math> and <math>C=(2,0).</math> The given condition <math>D=(7,5)</math> verifies our guess, as shown below.
 +
<asy>
 +
/* Made by MRENTHUSIASM */
 +
size(200);
  
When a line segment hits and bounces off a coordinate axis at point <math>P,</math> the ray entering <math>P</math> and the ray leaving <math>P</math> have negative slopes. <i><b>Geometrically, these two rays coincide when reflected about the line perpendicular to that coordinate axis, creating line symmetry.</b></i> Let the slope of <math>\overline{AB}</math> be <math>m.</math> It follows that the slope of <math>\overline{BC}</math> is <math>-m,</math> and the slope of <math>\overline{CD}</math> is <math>m.</math> Here, we conclude that <math>\overline{AB}\parallel\overline{CD}.</math>
+
int xMin = -3;
 +
int xMax = 9;
 +
int yMin = -3;
 +
int yMax = 7;
  
Next, we locate <math>E</math> on <math>\overline{CD}</math> such that <math>\overline{BE}\parallel\overline{AD},</math> from which <math>ABED</math> is a parallelogram, as shown below.  
+
//Draws the horizontal gridlines
[[File:2021 AMC 12A Problem 11 (3) LaTeX.png|center]]
+
void horizontalLines()
Graph in Desmos: https://www.desmos.com/calculator/lgfiiqgqc2
+
{
 +
  for (int i = yMin+1; i < yMax; ++i)
 +
  {
 +
    draw((xMin,i)--(xMax,i), mediumgray+linewidth(0.4));
 +
  }
 +
}
  
Let <math>B=(0,b).</math> In parallelogram <math>ABED,</math> we get <math>E=(4,b).</math> By symmetry, we obtain <math>C=(2,0).</math>
+
//Draws the vertical gridlines
 +
void verticalLines()
 +
{
 +
  for (int i = xMin+1; i < xMax; ++i)
 +
  {
 +
    draw((i,yMin)--(i,yMax), mediumgray+linewidth(0.4));
 +
  }
 +
}
  
Applying the slope formula to <math>\overline{AB}</math> and <math>\overline{DC}</math> gives <cmath>m=\frac{5-b}{3-0}=\frac{5-0}{7-2}.</cmath> Equating the last two expressions gives <math>b=2.</math>
+
//Draws the horizontal ticks
 +
void horizontalTicks()
 +
{
 +
  for (int i = yMin+1; i < yMax; ++i)
 +
  {
 +
    draw((-3/16,i)--(3/16,i), black+linewidth(1));
 +
  }
 +
}
  
By the Distance Formula, <math>AB=3\sqrt2,BC=2\sqrt2,</math> and <math>CD=5\sqrt2.</math> The total distance that the beam will travel is <cmath>AB+BC+CD=\boxed{\textbf{(C) }10\sqrt2}.</cmath>
+
//Draws the vertical ticks
 +
void verticalTicks()
 +
{
 +
  for (int i = xMin+1; i < xMax; ++i)
 +
  {
 +
    draw((i,-3/16)--(i,3/16), black+linewidth(1));
 +
  }
 +
}
  
~MRENTHUSIASM
+
horizontalLines();
 +
verticalLines();
 +
horizontalTicks();
 +
verticalTicks();
 +
draw((xMin,0)--(xMax,0),black+linewidth(1.5),EndArrow(5));
 +
draw((0,yMin)--(0,yMax),black+linewidth(1.5),EndArrow(5));
 +
label("$x$",(xMax,0),(2,0));
 +
label("$y$",(0,yMax),(0,2));
  
==Solution 4 (Answer Choices and Educated Guesses)==
+
pair A = (3,5);
Define points <math>A,B,C,</math> and <math>D</math> as Solution 2 does.
+
pair B = (0,2);
 +
pair C = (2,0);
 +
pair D = (7,5);
  
Since choices <math>\textbf{(B)}, \textbf{(C)},</math> and <math>\textbf{(D)}</math> all involve <math>\sqrt2,</math> we suspect that one of them is the correct answer. We take a guess in faith that <math>\overline{AB},\overline{BC},</math> and <math>\overline{CD}</math> all form <math>45^\circ</math> angles with the coordinate axes, from which <math>B=(0,2)</math> and <math>C=(2,0).</math> The given condition <math>D=(7,5)</math> verifies our guess. Following the penultimate paragraph of Solution 3 gives the answer <math>\boxed{\textbf{(C) }10\sqrt2}.</math>
+
draw(A--B--C--D,red);
 +
dot(A,linewidth(3.5));
 +
dot(B,linewidth(3.5));
 +
dot(C,linewidth(3.5));
 +
dot(D,linewidth(3.5));
 +
label("$A(3,5)$",A,(0,2),UnFill);
 +
label("$B$",B,(-2,0),UnFill);
 +
label("$C$",C,(0,-2),UnFill);
 +
label("$D(7,5)$",D,(0,2),UnFill);
 +
</asy>
 +
Following the last paragraph of Solution 2 gives the answer <math>\boxed{\textbf{(C) }10\sqrt2}.</math>
  
 
~MRENTHUSIASM
 
~MRENTHUSIASM
 +
 +
==Solution 4 (System of Linear Equations)==
 +
Denote <math>(3, 5)</math> as point <math>P</math> and <math>(7, 5)</math> as point <math>S</math>. Define the point <math>Q</math> on the <math>y</math>-axis that the laser hits as <math>(0, y_1)</math> and the point <math>R</math> on the <math>x</math>-axis that the laser hits as <math>(x_1,0)</math>. The laser will bounce off of the two axes at right angles, so we have that line <math>PQ \parallel RS</math>, meaning they have the same slope which we will denote as <math>k</math>. Line <math>QR</math> will be perpendicular to both <math>PQ</math> and <math>RS</math>, meaning it will have slope <math>-\frac{1}{k}</math>.
 +
 +
We can write the equations for lines <math>PQ</math> and <math>RS</math> in point-slope form, and then plug in points <math>Q</math> and <math>R</math> into those respective equations to get equations in terms of our three variables. We can also write the equation for line <math>QR</math> in slope-intercept form and plug in point <math>R</math>. Doing this yields
 +
<cmath>\begin{align*}
 +
    y_1-5&=k(0-3)\\
 +
    0-5&=k(x_1-7)\\
 +
    0&=-\frac{1}{k}x_1+y_1.
 +
\end{align*}</cmath>
 +
We can simplify this into the following cubic equation in terms of <math>k</math>:
 +
<cmath>-3k^3+5k^2-7k+5=0.</cmath>
 +
Luckily, we notice that the coefficients sum up to zero, so <math>k=1</math>. Now we can solve for the other two, yielding <math>x_1=y_1=2</math>. Now we simply have three <math>45^{\circ}</math>-<math>45^{\circ}</math>-<math>90^{\circ}</math> triangles with leg lengths of <math>3, 2,</math> and <math>5</math>, so our final distance is <cmath>3\sqrt{2}+2\sqrt{2}+5\sqrt{2}=\boxed{\textbf{(C) }10\sqrt2}.</cmath>
 +
~Mooshiros
  
 
== Video Solution by OmegaLearn (Using Reflections and Distance Formula) ==
 
== Video Solution by OmegaLearn (Using Reflections and Distance Formula) ==
Line 71: Line 238:
  
 
~IceMatrix
 
~IceMatrix
 +
 +
==Video Solution (Quick and Easy)==
 +
https://youtu.be/Hw-mYGUvflQ
 +
 +
~Education, the Study of Everything
  
 
==See also==
 
==See also==
 
{{AMC12 box|year=2021|ab=A|num-b=10|num-a=12}}
 
{{AMC12 box|year=2021|ab=A|num-b=10|num-a=12}}
 
{{MAA Notice}}
 
{{MAA Notice}}

Latest revision as of 01:28, 1 November 2023

Problem

A laser is placed at the point $(3,5)$. The laser beam travels in a straight line. Larry wants the beam to hit and bounce off the $y$-axis, then hit and bounce off the $x$-axis, then hit the point $(7,5)$. What is the total distance the beam will travel along this path?

$\textbf{(A) }2\sqrt{10} \qquad \textbf{(B) }5\sqrt2 \qquad \textbf{(C) }10\sqrt2 \qquad \textbf{(D) }15\sqrt2 \qquad \textbf{(E) }10\sqrt5$

Diagram

[asy] /* Made by MRENTHUSIASM */ size(200);   int xMin = -3; int xMax = 9; int yMin = -3; int yMax = 7;  draw((xMin,0)--(xMax,0),black+linewidth(1.5),EndArrow(5)); draw((0,yMin)--(0,yMax),black+linewidth(1.5),EndArrow(5)); label("$x$",(xMax,0),(2,0)); label("$y$",(0,yMax),(0,2));  pair A = (3,5); pair B = (0,2); pair C = (2,0); pair D = (7,5);  draw(A--B--C--D,red); dot(A,linewidth(3.5)); dot(B,linewidth(3.5)); dot(C,linewidth(3.5)); dot(D,linewidth(3.5)); label("$(3,5)$",A,(0,2)); label("$(7,5)$",D,(0,2)); [/asy] ~MRENTHUSIASM

Solution 1 (Reflections)

Let $A=(3,5)$ and $D=(7,5).$ Suppose that the beam hits and bounces off the $y$-axis at $B,$ then hits and bounces off the $x$-axis at $C.$

When the beam hits and bounces off a coordinate axis, the angle of incidence and the angle of reflection are congruent. Therefore, we straighten up the path of the beam by reflections:

  1. We reflect $\overline{BC}$ about the $y$-axis to get $\overline{BC'}.$
  2. We reflect $\overline{CD}$ about the $x$-axis to get $\overline{C'D'}$ with $D'=(7,-5),$ then reflect $\overline{C'D'}$ about the $y$-axis to get $\overline{C'D''}$ with $D''=(-7,-5).$

We obtain the following diagram: [asy] /* Made by MRENTHUSIASM */ size(225);   int xMin = -9; int xMax = 9; int yMin = -7; int yMax = 7;  draw((xMin,0)--(xMax,0),black+linewidth(1.5),EndArrow(5)); draw((0,yMin)--(0,yMax),black+linewidth(1.5),EndArrow(5)); label("$x$",(xMax,0),(2,0)); label("$y$",(0,yMax),(0,2));  pair A = (3,5); pair B = (0,2); pair C = (2,0); pair D = (7,5); pair E = (-2,0); pair F = (7,-5); pair G = (-7,-5);  draw(A--B--C--D,red); draw(B--E,heavygreen+dashed); draw(C--F,heavygreen+dashed); draw(E--G,heavygreen+dashed); dot("$A(3,5)$",A,(0,2),linewidth(3.5)); dot("$B$",B,(-2,0),linewidth(3.5)); dot("$C$",C,(0,-2),linewidth(3.5)); dot("$D(7,5)$",D,(0,2),linewidth(3.5)); dot("$C'$",E,(0,-2),linewidth(3.5)); dot("$D'(7,-5)$",F,(0,-2),linewidth(3.5)); dot("$D''(-7,-5)$",G,(0,-2),linewidth(3.5)); [/asy] The total distance that the beam will travel is \begin{align*} AB+BC+CD&=AB+BC+CD' \\ &=AB+BC'+C'D'' \\ &=AD'' \\ &=\sqrt{((3-(-7))^2+(5-(-5))^2} \\ &=\boxed{\textbf{(C) }10\sqrt2}. \end{align*} ~MRENTHUSIASM (Solution)

~JHawk0224 (Proposal)

Solution 2 (Parallelogram)

Define points $A,B,C,$ and $D$ as Solution 1 does. Moreover, let $E$ be a point on $\overline{CD}$ such that $\overline{BE}$ is perpendicular to the $y$-axis, and $F$ be a point on $\overline{BE}$ such that $\overline{CF}$ is perpendicular to the $x$-axis, as shown below. [asy] /* Made by MRENTHUSIASM */ size(200);   int xMin = -3; int xMax = 9; int yMin = -3; int yMax = 7;  draw((xMin,0)--(xMax,0),black+linewidth(1.5),EndArrow(5)); draw((0,yMin)--(0,yMax),black+linewidth(1.5),EndArrow(5)); label("$x$",(xMax,0),(2,0)); label("$y$",(0,yMax),(0,2));  pair A = (3,5); pair B = (0,2); pair C = (2,0); pair D = (7,5); pair E = (4,2); pair F = (2,2);  draw(A--B--C--D,red); draw(A--D^^B--E^^C--F,heavygreen+dashed); dot("$A(3,5)$",A,(0,2),linewidth(3.5)); dot("$B$",B,(-2,0),linewidth(3.5)); dot("$C$",C,(0,-2),linewidth(3.5)); dot("$D(7,5)$",D,(0,2),linewidth(3.5)); dot("$E$",E,(2,0),linewidth(3.5)); dot("$F$",F,(0,1.5),linewidth(3.5)); [/asy] When the beam hits and bounces off a coordinate axis, the angle of incidence and the angle of reflection are congruent, from which $\angle ABF=\angle CBF$ and $\angle BCF=\angle ECF.$ We conclude that $\triangle BCF\cong\triangle ECF$ by ASA, so $\angle CBF=\angle CEF.$ It follows that $\angle ABF=\angle CEF$ by transitive, so $\overline{AB}\parallel\overline{CD}$ by the Converse of the Alternate Interior Angles Theorem.

Note that $\overline{AD}\parallel\overline{BE}.$ Since the opposite sides are parallel, quadrilateral $ABED$ is a parallelogram. From $\triangle BCF\cong\triangle ECF,$ we get $BF=EF=2,$ so $C=(2,0).$

Let $B=(0,b).$ We equate the slopes of $\overline{AB}$ and $\overline{DC}:$ \[\frac{5-b}{3-0}=\frac{5-0}{7-2},\] from which $b=2,$ or $B=(0,2).$

By the Distance Formula, we have $AB=3\sqrt2,BC=2\sqrt2,$ and $CD=5\sqrt2.$ The total distance that the beam will travel is \[AB+BC+CD=\boxed{\textbf{(C) }10\sqrt2}.\]

Remark

When a straight line hits and bounces off a coordinate axis at point $P,$ the ray entering $P$ and the ray leaving $P$ always have negative slopes. In this problem, $\overline{AB}$ and $\overline{BC}$ have negative slopes; $\overline{BC}$ and $\overline{CD}$ have negative slopes. So, $\overline{AB}$ and $\overline{CD}$ have the same slope, or $\overline{AB}\parallel\overline{CD}.$

~MRENTHUSIASM

Solution 3 (Educated Guess)

Define points $A,B,C,$ and $D$ as Solution 1 does.

Since choices $\textbf{(B)}, \textbf{(C)},$ and $\textbf{(D)}$ all involve $\sqrt2,$ we suspect that one of them is the correct answer. We take a guess in faith that $\overline{AB},\overline{BC},$ and $\overline{CD}$ all form $45^\circ$ angles with the coordinate axes, from which $B=(0,2)$ and $C=(2,0).$ The given condition $D=(7,5)$ verifies our guess, as shown below. [asy] /* Made by MRENTHUSIASM */ size(200);   int xMin = -3; int xMax = 9; int yMin = -3; int yMax = 7;  //Draws the horizontal gridlines void horizontalLines() {   for (int i = yMin+1; i < yMax; ++i)   {     draw((xMin,i)--(xMax,i), mediumgray+linewidth(0.4));   } }  //Draws the vertical gridlines void verticalLines() {   for (int i = xMin+1; i < xMax; ++i)   {     draw((i,yMin)--(i,yMax), mediumgray+linewidth(0.4));   } }  //Draws the horizontal ticks void horizontalTicks() {   for (int i = yMin+1; i < yMax; ++i)   {     draw((-3/16,i)--(3/16,i), black+linewidth(1));   } }  //Draws the vertical ticks void verticalTicks() {   for (int i = xMin+1; i < xMax; ++i)   {     draw((i,-3/16)--(i,3/16), black+linewidth(1));   } }  horizontalLines(); verticalLines(); horizontalTicks(); verticalTicks(); draw((xMin,0)--(xMax,0),black+linewidth(1.5),EndArrow(5)); draw((0,yMin)--(0,yMax),black+linewidth(1.5),EndArrow(5)); label("$x$",(xMax,0),(2,0)); label("$y$",(0,yMax),(0,2));  pair A = (3,5); pair B = (0,2); pair C = (2,0); pair D = (7,5);  draw(A--B--C--D,red); dot(A,linewidth(3.5)); dot(B,linewidth(3.5)); dot(C,linewidth(3.5)); dot(D,linewidth(3.5)); label("$A(3,5)$",A,(0,2),UnFill); label("$B$",B,(-2,0),UnFill); label("$C$",C,(0,-2),UnFill); label("$D(7,5)$",D,(0,2),UnFill); [/asy] Following the last paragraph of Solution 2 gives the answer $\boxed{\textbf{(C) }10\sqrt2}.$

~MRENTHUSIASM

Solution 4 (System of Linear Equations)

Denote $(3, 5)$ as point $P$ and $(7, 5)$ as point $S$. Define the point $Q$ on the $y$-axis that the laser hits as $(0, y_1)$ and the point $R$ on the $x$-axis that the laser hits as $(x_1,0)$. The laser will bounce off of the two axes at right angles, so we have that line $PQ \parallel RS$, meaning they have the same slope which we will denote as $k$. Line $QR$ will be perpendicular to both $PQ$ and $RS$, meaning it will have slope $-\frac{1}{k}$.

We can write the equations for lines $PQ$ and $RS$ in point-slope form, and then plug in points $Q$ and $R$ into those respective equations to get equations in terms of our three variables. We can also write the equation for line $QR$ in slope-intercept form and plug in point $R$. Doing this yields \begin{align*}     y_1-5&=k(0-3)\\     0-5&=k(x_1-7)\\     0&=-\frac{1}{k}x_1+y_1. \end{align*} We can simplify this into the following cubic equation in terms of $k$: \[-3k^3+5k^2-7k+5=0.\] Luckily, we notice that the coefficients sum up to zero, so $k=1$. Now we can solve for the other two, yielding $x_1=y_1=2$. Now we simply have three $45^{\circ}$-$45^{\circ}$-$90^{\circ}$ triangles with leg lengths of $3, 2,$ and $5$, so our final distance is \[3\sqrt{2}+2\sqrt{2}+5\sqrt{2}=\boxed{\textbf{(C) }10\sqrt2}.\] ~Mooshiros

Video Solution by OmegaLearn (Using Reflections and Distance Formula)

https://youtu.be/e7tNtd-fgeo

~ pi_is_3.14

Video Solution by Hawk Math

https://www.youtube.com/watch?v=AjQARBvdZ20

Video Solution by TheBeautyofMath

https://youtu.be/ySWSHyY9TwI

~IceMatrix

Video Solution (Quick and Easy)

https://youtu.be/Hw-mYGUvflQ

~Education, the Study of Everything

See also

2021 AMC 12A (ProblemsAnswer KeyResources)
Preceded by
Problem 10
Followed by
Problem 12
1 2 3 4 5 6 7 8 9 10 11 12 13 14 15 16 17 18 19 20 21 22 23 24 25
All AMC 12 Problems and Solutions

The problems on this page are copyrighted by the Mathematical Association of America's American Mathematics Competitions. AMC logo.png